LSAT and Law School Admissions Forum

Get expert LSAT preparation and law school admissions advice from PowerScore Test Preparation.

 ylikate
  • Posts: 30
  • Joined: Aug 27, 2013
|
#10891
Have a few questions regarding this comparative passage set

Q4: Why is (A) incorrect? I thought wind which is an alt explanation for the pattern weakens the argument for water. Also why is (C) correct?


Thank you in advance!
 Steve Stein
PowerScore Staff
  • PowerScore Staff
  • Posts: 1153
  • Joined: Apr 11, 2011
|
#10895
Answer choice A provides that "many" of the patterns could have been the result of wind; the author of passage A mentions at the end of paragraph three that some of the patterns could have resulted from wind, but that others provide reliable evidence of flowing water, so this would not weaken the argument.

Correct answer choice C provides that Mars' known liquids could have created the layers in the rock--the "crossbedding" and "festooning" discussed at the beginning of the third paragraph. this fact, if discovered, would call that evidence into question, thus weakening the author's argument.

I hope that's helpful! Please let me know whether this is clear--thanks!

~Steve
 Blueballoon5%
  • Posts: 156
  • Joined: Jul 13, 2015
|
#27837
Question:

4.) I am a bit confused why answer choice C is correct. When this answer choice refers to "liquids", does this suggest that liquids =/= water? And how would this weaken the author's argument? It seems to me that answer choice B is a better answer. Wouldn't the author's argument weaken if experts, who study the chemical composition, disagree with the author's conclusion?

Thanks!!
 Shannon Parker
PowerScore Staff
  • PowerScore Staff
  • Posts: 147
  • Joined: Jun 08, 2016
|
#27838
hi there,

The argument advanced in passage A is that experts have concluded that water at some point existed on Mars, based largely on the fact that there are patterns in the Martian rock which the experts believe was caused by water. Question 4 asks "which of the following if discovered would most weaken" this argument? Answer choice (C) states that "many liquids known to have existed on Mars could have caused the patterns which have been noted in layers of Martian rock." If the experts main support for the existence of water is that they caused these patterns, but it turns out that there were several liquids other than water on the planet, and any of them were capable of causing the patterns, it becomes more likely that one of those liquids caused the patterns instead of water, thus hurting the argument.

Answer choice B simply states that other people studying the evidence disagree with the conclusion. In weaken questions you want to look for answer choices that have an effect on the reasoning of the argument.

I hope this helps.

~Shannon

Get the most out of your LSAT Prep Plus subscription.

Analyze and track your performance with our Testing and Analytics Package.